« Vrati se
Let a_{1},a_{2},\ldots ,a_{n} be positive real numbers such that a_{1}+a_{2}+\cdots +a_{n}<1. Prove that

\frac{a_{1} a_{2} \cdots a_{n} \left[ 1 - (a_{1} + a_{2} + \cdots + a_{n}) \right] }{(a_{1} + a_{2} + \cdots + a_{n})( 1 - a_1)(1 - a_2) \cdots (1 - a_n)} \leqslant \frac{1}{n^{n+1}}

Slični zadaci

#NaslovOznakeRj.KvalitetaTežina
2066IMO Shortlist 2001 problem A29
2206IMO Shortlist 2006 problem A31
2207IMO Shortlist 2006 problem A43
2266IMO Shortlist 2008 problem A36
2291IMO Shortlist 2009 problem A218
2351Mala olimpijada 1998 zadatak 32